Nelson Functions 11
Nelson Functions 11
1st Edition
Chris Kirkpatrick, Marian Small
ISBN: 9780176332037
Textbook solutions

All Solutions

Page 202: Practice Questions

Exercise 1
Step 1
1 of 4
$bold{Concept:}$ For the quadratic function in vertex form $f(x)=a(x-h)^2+k$

direction of opening:

$a>0implies$ up
$a0implies { f(x)inbold{R};|;f(x)geq k}$

if $a<0implies { f(x)inbold{R};|;f(x)leq k}$

Step 2
2 of 4
$bold{Solution:}$

a) Since $a=-3<0$, it opens downward. The vertex is $(2,5)$ and axis of symmetry is $x=2$

b) domain: ${ xin bold{R}}$

range: ${ f(x)inbold{R};|;f(x) leq 5}$

Step 3
3 of 4
c) You can evaluate $f(x)$ for $x=0,1,2,3,4$ to obtain data points and then sketch the graph.

Exercise scan

Result
4 of 4
a) down, vertex $(2,5)$ , $x=2$

b) domain = ${ xin bold{R}}$ , range = ${ yin bold{R};|;yleq 5}$

c) see graph inside

Exercise 2
Step 1
1 of 4
$bold{Concept:}$ For the quadratic function in factored form $f(x)=a(x-r)(x-s)$

zeros: $r$ and $s$

direction of opening:

$a>0implies$ up
$a0implies { f(x)inbold{R};|;f(x)geq k}$

if $a<0implies { f(x)inbold{R};|;f(x)leq k}$

Step 2
2 of 4
$bf{Solution:}$

a) Since $a=4>0$, it opens upwards and it has zeros at $x=2$ and $-6$.

The value of $x$ at the vertex corresponds to the average of two zeros.

$h=dfrac{2+(-6)}{2}=-2$

$k=f(-2)=4(-2-2)(-2+6)=-64$

and axis of symmetry is $x=-2$

b) domain: ${ xin bold{R}}$

range: ${ f(x)inbold{R};|;f(x) geq -64}$

Step 3
3 of 4
c) You can plot the $x$-intercepts $(-6,0)$ and $(2,0)$ and the vertex$(-2,64)$ then draw a smooth curve.

Exercise scan

Result
4 of 4
a) up, zeros 2 and $-6$

b) vertex $(-2,-64)$

c) domain: ${ xinbold{R}}$ ; range : ${ yin bold{R};|;ygeq -64}$

Exercise 3
Step 1
1 of 3
$bold{Concept:}$ If $(x_1,y)$ and $(x_2,y)$ are two points on a parabola with equal value of $y$, then the axis of symmetry is $x=dfrac{x_1+x_2}{2}$.
Step 2
2 of 3
$bf{Solution:}$

The two given points $(-5,3)$ and $(3,3)$ has the same value of $y$-coordinate, thus, the average of the $x$-coordinates must correspond to the axis of symmetry.

axis of symmetry: $x=dfrac{-5+3}{2}=dfrac{-2}{2}=-1$

Result
3 of 3
$$
x=-1
$$
Exercise 4
Step 1
1 of 4
$bold{Concept:}$ The maximum or minimum value will occur at the vertex. If $a>0$, the vertex corresponds to the minimum value while if $a<0$, the vertex corresponds to the maximum value.
Step 2
2 of 4
$bold{Solution:}$

a) In vertex form $f(x)=a(x-h)^2+k$

The vertex is at $(h,k)$

In this case, $a=-3<0$,

so the maximum occurs at $(4,7)$

The maximum value is $7$ which occurs at $x=4$

Step 3
3 of 4
b) In factored form $f(x)=a(x-r)(x-s)$

vertex$(h,k)$ where $h=dfrac{r+s}{2}$ and $k=f(h)$

In this case,

$h=dfrac{0+(-6)}{2}=-3$

$k=f(-3)=4(-3)(-3+6)=-36$

With $a=4>0$, the minimum occurs at $(-3,-36)$

The minimum value is $-36$ which occurs at $x=-3$

Result
4 of 4
a) The maximum value is $7$ which occurs at $x=4$

b) The minimum value is $-36$ which occurs at $x=-3$

Exercise 5
Step 1
1 of 3
$bold{Concept:}$ The maximum or minimum value of a quadratic function occurs at its vertex. For the standard form

$f(x)=ax^2+bx+c$

the vertex is $(h,k)$ where

$h=-dfrac{b}{2a}$

$$
k=f(h)
$$

Step 2
2 of 3
$bold{Solution:}$ Rewrite the function in standard form

$h(t)=-4.9t^2+28t+2$

$a=-4.9$ , $b=28$ , $c=2$

Since $a<0$, the vertex corresponds to the maximum.

We shall find the coordinates of the vertex to find the maximum height

$h=-dfrac{b}{2a}=-dfrac{28}{2(-4.9)}=2.86$

$k=f(2.86)=-4.9(2.86)^2+28(2.86)+2=42$ m

Therefore, the maximum height is $42$ m which occurs at about $2.9$ s

Result
3 of 3
The maximum height is $42$ m which occurs at about $2.9$ s
Exercise 6
Step 1
1 of 2
To find the the inverse of $y=x^2$,

Replace $y$ with $x$ and replace $x$ with $y$ (swap the variables), then solve for $y$ in terms of $x$

$x=y^2implies y=pmsqrt{x}$

Therefore, $g(x)=sqrt{x}$ and $h(x)=-sqrt{x}$ are two branches of the inverse of $f(x)=x^2$

Result
2 of 2
$g(x)=sqrt{x}$ and $h(x)=-sqrt{x}$ are two branches of the inverse of $f(x)=x^2$
Exercise 7
Step 1
1 of 3
Remember that functions must pass the “vertical line test”, that is, it should touch any vertical line only once.

The inverse of a quadratic function is a parabola that opens either left or right. It has two branches and it would touch the vertical line twice. Thus, it is not function.

Note that if the domain of the quadratic function is restricted to a single branch of the parabola, then its inverse would be a function since it will only have once branch.

Step 2
2 of 3
For example, the inverse of the quadratic function $f(x)=x^2$ is $g(x)=pmsqrt{x}$. However, $g(x)$ fails the vertical line test.

Exercise scan

Result
3 of 3
No. The inverse of a quadratic function is parabola that opens to the left or right and has upper and lower branch which fails the vertical line test.
Exercise 8
Step 1
1 of 4
a) From the graph, the vertex is $(2,4)$ so we can write the vertex form as

$f(x)=a(x-2)^2+4$

To find $a$, we can use any point on the parabola. We will choose $(1,2)$

$2=a(1-2)^2+4implies a=dfrac{2-4}{(1-2)^2}=-2$

$f(x)=y=-2(x-2)^2+4$

Now, we will find the inverse of this function by swapping $x$ and $y$

$x=-2(y-2)^2+4$

$(y-2)^2=dfrac{x-4}{-2}$

$y-2=pmsqrt{dfrac{4-x}{2}}$

$$
y=2pmsqrt{dfrac{4-x}{2}}
$$

Step 2
2 of 4
From the graph, we see the following points $(0,-4)$ , $(1,2)$ , $(2,4)$ , $(3,2)$ , $(4,-4)$

We can sketch the inverse function by swapping the coordinates. Thus, the inverse shall contain the following points.

$(-4,0)$, $(2,1)$ , $(4,2)$, $(2,3)$ , $(-4,4)$

Exercise scan

Step 3
3 of 4
b) From the graph, the domain of the inverse relation is the set of $x$-values not more than $4$

domain = ${ xinbold{R};|;xleq 4}$

The range is the set of all real numbers.

range = ${ yinbold{R}}$

c) The inverse relation cannot pass the vertical line test. For example, the vertical line at $x=-4$ would touch the points $(-4,0)$ and $(-4,4)$, thus, it is not a function.

Result
4 of 4
a) see graph inside

b) domain = ${ xinbold{R};|;xleq 4}$ ; range = ${ yinbold{R}}$

c) not a function ; does not pass vertical line test

Exercise 9
Step 1
1 of 2
In the following, remember that $sqrt{a^2b}=asqrt{b}$

a) $sqrt{98}$
$=sqrt{49times 2}$
$=sqrt{7^2times 2}$
$=7sqrt{2}$

b) $-5sqrt{32}$
$=-5sqrt{16times 2}$
$=-5sqrt{4^2times 2}$
$=(-5times 4)sqrt{2}$
$=-20sqrt{2}$

c) $4sqrt{12}-3sqrt{48}$
$=4sqrt{4times 3}-3sqrt{16times 3}$
$=4sqrt{2^2times 3}-3sqrt{4^2times 3}$
$=8sqrt{3}-12sqrt{3}$
$=-4sqrt{3}$

d) $(3-2sqrt{7})^2$
$=3^2-2(3)(-2sqrt{7})+(2sqrt{7})^2$
$=9-12sqrt{7}+4times 7$

$$
=37-12sqrt{7}
$$

Result
2 of 2
a) $7sqrt{2}$

b) $-20sqrt{2}$

c) $-4sqrt{4}$

d) $37-12sqrt{7}$

Exercise 10
Step 1
1 of 2
Substitute the given values to the formula:

$a=5$, $b=7$ , $c=10$

$s=dfrac{1}{2}(a+b+c)=dfrac{1}{2}(5+7+10)=11$

$A=sqrt{s(s-a)(s-b)(s-c)}$

$=sqrt{11(11-5)(11-7)(11-10)}$

$=sqrt{11(6)(4)(1)}$

$=sqrt{264}$

$=sqrt{4times 66}$

$=sqrt{2^2times 66}$

$=2sqrt{66}$

The area of the triangle is $2sqrt{66}$ square units.

Result
2 of 2
$2sqrt{66}$ square units
Exercise 11
Step 1
1 of 3
In this case,

$a=6$ , $b=3$

$c=sqrt{6^2+3^2}$

$c=sqrt{36+9}$

$c=sqrt{45}$

$c=sqrt{9times 5}$

$c=sqrt{3^2times 5}$

$c=3sqrt{5}$

The hypotenuse $c$ of a triangle with legs $a$ and $b$ can be calculated using Pythagorean formula:

$$
c^2=a^2+b^2
$$

Step 2
2 of 3
$P=6+3+3sqrt{5}$

$P=9+3sqrt{5}$

Therefore, the perimeter of the triangle is

$P=9+3sqrt{5}$ cm

The perimeter of a triangle is the sum of all three sides.

$$
P=a+b+c
$$

Result
3 of 3
$P=9+3sqrt{5}$ cm
Exercise 12
Step 1
1 of 4
$bold{Concept:}$ The $x$-intercepts of a quadratic function are the points at which the graph touches the $x$-axis or when $f(x)=0$

If $x_1$ and $x_2$ are the zeros of the quadratic function, then the $x$-intercepts are

$(x_1,0)$ and $(x_2,0)$

Step 2
2 of 4
$bold{Solution:}$ We can solve this by factoring

$f(x)=2x^2+x-15=0$

$(2x-5)(x+3)=0$

$x=dfrac{5}{2}$ or $-3$

Therefore, the $x$-intercepts are

$left(dfrac{5}{2},0right)$ and $(-3,0)$

Step 3
3 of 4
We can confirm this by graphingExercise scan
Result
4 of 4
$left(dfrac{5}{2},0right)$ and $(-3,0)$
Exercise 13
Step 1
1 of 5
a) First find $t$ at year $2020$

$t=2020-2007=13$

Now substitute this value of $t$ to the population model

$P(t)=12t^2+800t+40;000$

$P(13)=12(13)^2+800(13)+40;000$

$P(13)=52;428$

Step 2
2 of 5
b) We shall find the value of $t$ such that $P(t)=300;000$

$300;000=12t^2+800t+40;000$

$12t^2+800t-260;000=0$

We will use quadratic formula

$a=12$ , $b=800$ , $c=-260;000$

$t=dfrac{-bpmsqrt{b^2-4ac}}{2a}$

$t=dfrac{-800pmsqrt{800^2-4(12)(-260;000)}}{2(12)}$

$t=-184.25$ or $117.59$

We can confirm this by graphing.

Exercise scan

Step 3
3 of 5
Since $t=0$ at year 2007.

$t=-184.25implies text{ year }=2007-184.25=1823$

$t=117.59implies text{ year } = 2007+117.59=2125$

Thus, the population is predicted to be $300,000$ in 1823 and 2125.

Step 4
4 of 5
$bold{Note:}$ The answer in your textbook is $1990$. This is obtained when the predicted population is 30,000 and not $300,000$. In this case,

$12t^2+800t+40;000=30;000$

$12t^2+800t+10;000=0$

$a=12$ , $b=800$ , $c=10;000$

$x=dfrac{-bpmsqrt{b^2-4ac}}{2a}$

$x=dfrac{-800pmsqrt{800^2-4(12)(10;000)}}{2(12)}$

$x=-50implies 2007-50=19567$

$-$16.67 $implies 2007-16.67=1990.33$

This occurs in year 1957 and 1990.

Exercise scan

Result
5 of 5
The population is predicted to be 30 000 in 1990 and 1957

The population is predicted to be 300 000 in 1823 and 2125

Exercise 14
Step 1
1 of 3
Let $x$ be the length of the rectangular garden and $w$ be the width.

Exercise scan

Step 2
2 of 3
We need to express the width in terms of the length $x$

Since the perimeter is $400$ m, we can write

$2x+2w=400$

$x+w=200implies w=200-x$

The area of the triangle is length $times$ width

$x(200-x)=8000$

$200x-x^2=8000$

$200x-x^2-8000=0$

$-x^2+200x-8000=0$

Solve by quadratic formula

$a=-1$ , $b=200$ , $c=-8000$

$x=dfrac{-bpmsqrt{b^2-4ac}}{2a}$

$x=dfrac{-200pmsqrt{(-200)^2-4(-1)(-8000)}}{2(-1)}$

$x=55.28$ or $144.72$

$w=200-55.28=144.72$

Therefore, both values of $x$ would lead to rectangle with dimensions

$55.28$ m by $144.72$ m

Result
3 of 3
$55.28$ m by $144.72$ m
Exercise 15
Step 1
1 of 2
To determine whether $h(t)$ can have a value of $9$, we need to find range of the function.

Remember that for parabola opening downwards $a<0$, the range is $h(t)leq k$

$h(t)=14t-5t^2$

$a=-5$ , $b=14$, $c=0$

The value of $t$ that corresponds to the vertex is

$t=-dfrac{b}{2a}=-dfrac{14}{2(-5)}=-1.4$

Now we shall evaluate $h(t)$ at $t=-1.4$ to find $k$

$k=h(-1.4)=14(1.4)-5(1.4)^2=9.8$

Therefore, the maximum height is $9.8$, thus, it can reach a height of $9$ m.

Result
2 of 2
Yes.
Exercise 16
Step 1
1 of 4
To determine the number of zeros of a quadratic function in standard form, calculate $b^2-4ac$

$f(x)=ax^2+bx+c$

$b^2-4ac>0implies$ 2 zeros

$b^2-4ac=0implies$ 1 zero

$b^2-4ac0$

$(2k)^2+2(2k)(-3)+(-3)^2-16>0$

$4k^2-12k+9-16>0$

$4k^2-12k-7>0$

$(2k-7)(2k+1)>0$

$k=dfrac{7}{2}$ , $-dfrac{1}{2}$

This is a parabola that opens upward so the function would be greater than zero for values that are on the left of $-dfrac{1}{2}$ and to the right of $dfrac{7}{2}$

$kdfrac{7}{2}$

Step 2
2 of 4
Example where $k<-dfrac{1}{2}$ is $k=-1$

$f(x)=4x^2-3x+2(-1)x+1$

$f(x)=4x^2-5x+1$

Exercise scan

Step 3
3 of 4
Example where $k>dfrac{7}{2}$ is $k=4$

$a=4$ , $b=2(4)-3=5$ , $c=1$

$$
y=4x^2+5x+1
$$

Exercise scan

Result
4 of 4
$x3.5$
Exercise 17
Step 1
1 of 3
$bold{Concept:}$ The break-even point of a profit function $P(x)$ are values of $x$ such that $P(x)=0$
Step 2
2 of 3
$bold{Solution:}$ We need to solve the equation

$P(x)=-2x^2+7x+8=0$

We shall use quadratic formula

$a=-2$ , $b=7$ , $c=8$

$x=dfrac{-bpmsqrt{b^2-4ac}}{2a}$

$x=dfrac{-7pmsqrt{7^2-4(-2)(8)}}{2(-2)}$

$x=dfrac{-7pmsqrt{113}}{-4}$

$x=dfrac{7pmsqrt{113}}{4}$

$x=-0.9075$ or $4.408$

Since the number of bikes $x$ (in thousands) cannot be negative, choose only $x=4.408$
which corresponds to $4408$ bikes.

Result
3 of 3
$4408$ bikes
Exercise 18
Step 1
1 of 3
$bold{Concept:}$ If a parabola has roots (zeros) $r$ and $s$, then the equation of parabola is

$y=a(x-r)(x-s)$

$y=a[x^2-(r+s)x+rs]$

To evaluate $a$, substitute any point on the parabola to the equation and solve for $a$

Step 2
2 of 3
$bold{Solution:}$ In this case, $r=2+sqrt{3}$ , $s=2-sqrt{3}$

$r+s=(2+sqrt{3})+(2-sqrt{3})=4$

$rs=(2+sqrt{3})(2-sqrt{3})=2^2-(sqrt{3})^2=4-3=1$

Thus, the equation becomes

$y=a(x^2-4x+1)$

We are given that $(2,5)$ is a point on the parabola,

$5=a(2^2-4(2)+1)$

$a=dfrac{5}{2^2-4(2)+1}=-dfrac{5}{3}$

Therefore, the equation of the parabola is

$y=-dfrac{5}{3}(x^2-4x+1)$

$$
y=-dfrac{5}{3}+dfrac{20}{3}x-dfrac{5}{3}
$$

Result
3 of 3
$$
y=-dfrac{5}{3}+dfrac{20}{3}x-dfrac{5}{3}
$$
Exercise 19
Step 1
1 of 3
begin{table}[]
defarraystretch{1.5}%
begin{tabular}{|l|l|}
hline
$bold{Form}$ & $bold{Family; of; Parabola}$ \ hline
begin{tabular}[c]{@{}l@{}}vertex form\ $f(x)=a(x-h)^2+k$end{tabular} & begin{tabular}[c]{@{}l@{}}if $a$ is varied\ $implies$ families with the same vertex and\ axis of symmetryend{tabular} \ hline
begin{tabular}[c]{@{}l@{}}factored form\ $f(x)=a(x-r)(x-s)$end{tabular} & begin{tabular}[c]{@{}l@{}}if $a$ is varied\ $implies$ families with the same $x$-intercepts\ and axis of symmetryend{tabular} \ hline
begin{tabular}[c]{@{}l@{}}standard form\ $f(x)=ax^2+bx+c$end{tabular} & begin{tabular}[c]{@{}l@{}}if $a$ and $b$ are varied\ $implies$ families with the same $y$-interceptend{tabular} \ hline
end{tabular}
end{table}
Step 2
2 of 3
This is in vertex form with varying $a$, thus it will have the same vertex and axis of symmetry.

Now, we shall evaluate $a$ by substituting the coordinates of the point $(-2,6)$

$f(x)=a(x+3)^2-4$

$6=a(-2+3)^2-4$

$a=dfrac{6+4}{(-2+3)^2}=10$

The member that passes through $(-2,6)$ is

$$
f(x)=10(x+3)^2-4
$$

Result
3 of 3
$$
f(x)=10(x+3)^2-4
$$
Exercise 20
Step 1
1 of 4
Interpret the given conditions:

parabolic arch $implies$ parabola that opens downward, we will arbitrarily choose the axis of symmetry of the parabola at $x=0$

$15$ m high $implies$ the vertex is $(0,15)$

$6$ m wide at a height of $8$ m $implies$ passes through $(-3,8)$ and $(3,8)$

Step 2
2 of 4
a) With vertex at $(0,15)$, the parabola must belong to the family

$y=a(x-0)^2+15$

We will find $a$ by substituting the point $(3,8)$

$8=a(3-0)^2+15$

$a=dfrac{8-15}{3^2}=-dfrac{7}{9}$

Therefore, the equation of the parabola is

$$
y=-dfrac{7}{9}x^2+15
$$

Step 3
3 of 4
b) The width of the base is distance between two zeros.

$-dfrac{7}{9}x^2+15=0$

$dfrac{7}{9}x^2=15$

$x=pmsqrt{dfrac{15}{7/9}}$

$x=pm 4.4$

Thus, the zeros are at $(-4.4,0)$ and $(4.4,0)$

The distance between these two points is $4.4-(-4.4)=8.8$

Therefore, the arch must be 8.8 m wide at the base.

Exercise scan

Result
4 of 4
a) $y=-dfrac{7}{9}x^2+15$

b) 8.8 m

Exercise 21
Step 1
1 of 6
$2x^2+4x-11=-3x+4$

$$
2x^2+7x-15=0
$$

Set the two equations equal to one another and put into standard form. Which is:

$$
ax^2+bx+c=0
$$

Step 2
2 of 6
$$
(x+5)(2x-3)
$$
Solve for $x$. This can most easily be done by factoring. We know that $3(5)=15$ and that $2(5)-7=3$. Therefore
Step 3
3 of 6
$x=-5$

$$
2x-3=0rightarrow x=frac{3}{2}=1.5
$$

Solve for $x$. This will be the x values for the points of intersection
Step 4
4 of 6
$-3(1.5)+4=-.5$

$$
-3(-5)+4=19
$$

Plug the values of x into either one of the original two equations. This will be the y-values for the intersection points
Step 5
5 of 6
$(1.5,-0.5)$ and $(-5,19)$
Putting the x and y values together results in
Result
6 of 6
$(1.5,-0.5)$ and $(-5,19)$
Exercise 22
Step 1
1 of 2
To find the intersection point of the baseball and the paintball, equate $h(t)=g(t)$

$h(t)=g(t)$

$-5t^2+20t+15=3t+3$

$-5t^2+17t+12=0$

$a=-5$, $b=17$, $c=12$

We shall first determine how many zeros are possible

$b^2-4ac=(17)^2-4(-5)(12)=529>0$

Since $b^2-4ac>0$, there will be two solutions.

We shall then use the quadratic formula

$t=dfrac{-bpmsqrt{b^2-4ac}}{2a}$

$t=dfrac{-17pmsqrt{529}}{2(-5)}$

$t=-0.6$ or 4

Time can’t be negative, so choose $t=4$

The height at $t=4$ can be solved using either $h(t)$ or $g(t)$

$g(t)=3(4)+3=15$ m

Therefore, the paintball will hit the baseball after 4 s at a height of 15 m.

Result
2 of 2
Yes, after 4 s at a height of 15 m.
Exercise 23
Step 1
1 of 4
$bold{Concept:}$ The intersection point of a parabola $f(x)$ and a line $g(x)$ are the solutions of $f(x)=g(x)$

If no values of $x$ satisfies the equation $(b^2-4ac<0)$, then $f(x)$ and $g(x)$ does not intersect.

Step 2
2 of 4
$bold{Solution:}$ Equate $f(x)$ and $g(x)$

a) $f(x)=g(x)$

$x^2-6x+9=-3x-5$

$x^2-3x+14=0$

$a=1$ , $b=-3$ , $c=14$

$b^2-4ac=(-3)^2-4(1)(14)=-47<0$

Since $b^2-4ac<0$, the parabola and the line do not intersect.

Exercise scan

Step 3
3 of 4
b) Answers can vary. As we can observe from the graph, if we make the slope 3, then it would intersect the parabola at two points.Exercise scan
Result
4 of 4
a) No since $b^2-4ac$ of $f(x)-g(x)$ is negative.

b) $g(x)=3x-5$

unlock
Get an explanation on any task
Get unstuck with the help of our AI assistant in seconds
New